Auditing Problems1

February 9, 2018 | Author: Ronnel Tagalogon | Category: Leverage (Finance), Capital Structure, Internal Audit, Stocks, Audit
Share Embed Donate


Short Description

wqerf...

Description

Auditing Problems watitiw 1. Draft income statement of Raffles Inc. is as follows: Sales Cost of sales Opening inventory Purchases Closing inventory Gross profit Expenses Profit

P700,000 60,000 430,000 90,000

400,000 300,000 200,000 P100,000

How much is the correct profit after the following adjustments? (1) Closing inventory includes goods costing P20,000 which are expected to realize at P19,000. (2) A customer has taken legal action for damages of P50,000 against Raffles. The lawyer of Raffles has advised the customer that he has a 25% chance of success. (3) After the balance sheet date, a vehicle was damaged in an accident. The carrying amount of the vehicle was P6,000. It was not insured. (4) Raffles has sued one of its competitors for P60,000. The chance of Raffles winning the case is 75%. The outcome will be known in three months. A . B. C. D .

P159,000 P103,000 P99,000 P49,000

Solution: C is correct Unadjusted profit (1) Write down of inventory to its NRV (20,000 – 19,000) (2) Loss not probable – no adjustment (3) Non-adjusting event after the balance sheet date - no adjustment (4) Gain not virtuallycertain – no adjustment Adjusted profit

P100,000 (1,000) P99,000

Page 1 of 45

2. Zod Company reports the following information as of December 31: Sales revenue Cost of goods sold Operating expenses Unrealized gain on available-for-sale securities

P800,000 600,000 90,000 30,000

How much should the company report as total comprehensive income? A . B. C. D .

P80,000 P110,000 P140,000 P200,000

Solution: C is correct Sales revenue Less: Cost of goods sold Less: Operating expenses Amount taken to P&L Amount taken to OCI Total comprehensive income

P800,000 600,000 90,000 P110,000 30,000 P140,000

3. You are assigned to the audit of Canon Company who spent P12,000,000 during the current year developing its new software package. Of this amount, P4,000,000 was spent before it was at the application development stage and the package was only to be used internally. The package was completed during the year and is expected to have a four year useful life. Canon Company has a policy of taking a full-year's amortization in the first year. After the development stage, P50,000 was spent on training employees to use the program. What amount should Canon Company report as an expense for the current year? A . B. C. D .

P2,012,500 P2,050,000 P6,012,500 P6,050,000

Solution: D is correct Amount spent before the development stage Amount spent on training employees Amortization of the software package (12M – 4M) / 4 years Total expense to be recognized

P4,000,000 50,000 2,000,000 P6,050,000

4. You were assigned to audit the factory accounts of Ministops Corporation for the year ended December 31, 2012. The following data were gathered: Manufacturing cost totalled P900,000. Cost of goods manufactured was P800,000 of which factory overhead was 75% of direct labor. Overhead was 25% of total manufacturing cost. Beginning work-in process inventory was 60% of ending work-in-process Page 2 of 45

inventory. Manufacturing costs for the year ended December 31, 2012 submitted to you by the factory accountant were as follows: Raw materials used Direct labor Factory overhead Total

P400,000 275,000 225,000 P900,000

Assume cost percentage relationships as stated were correct. Which of the following adjusting journal entries is correct? A. Raw materials used Direct labor B. Direct labor Raw materials used C. Raw materials used Direct labor D. Direct labor Raw materials used

P25,000 P25,000 P25,000 P25,000 P50,000 P50,000 P50,000 P50,000

Solution: B is correct Factory overhead (75% of direct labor) Divided by 75% Direct labor, per recomputation Direct labor, unadjusted Adjustment to direct labor

P225,000 75% 300,000 275,000 P25,000

5. On June 1, 2008, Psy Corporation purchased as a long term investment 6,000 of the P1,000 face value, 8% bonds of Lyre Corporation. Psy Corporation has the positive intention and ability to hold these bonds to maturity. The bonds were purchased to yield 10% interest. Interest is payable semi-annually on December 1 and June 1. The bonds mature on June 1, 2014. On November 1, 2009, Psy Corporation sold the bonds for a total consideration of P5,887,500. Based on the above and the result of your audit, determine the following: (Round off present value factors to four decimal places) - the purchase price of the bonds on June 1, 2008 is A . B. C. D .

P5,467,992 P5,467,992 P5,545,104 P5,436,894

Solution: A is correct PV of the principal (0.5568 x 6,000,000) PV of the interest (8.8633 x 6,000,000 x 4%)

P3,340,800

2,127,192 P5,467,992 6. The Tiger Corporation included the following in its unadjusted trial balance as of December 31, 2012: Inventory, 12/31/11 Purchases

P19,450,000 127,850,000 Page 3 of 45

Additional information:  



The inventory at December 31, 2012 was counted at a cost of P8.5 million. This includes P500,000 of slow moving inventory that is expected to be sold for P300,000. Sales include P8 million for goods sold in December 2012 for cash to Beer Finance Company. The cost of these goods was P6 million. Beer Finance Company has the option to require Tiger to repurchase these goods within one month of year-end at their original selling price plus a facilitating fee of P250,000. The Company’s accounting policy is to present cost of writing down inventory to NRV as part of cost of sales.

The cost of sales for the year ended December 31, 2012 is A P138,800,000 . B. P133,000,000 C. P132,800,000 D P139.000.000 . Solution: B is correct Amount in thousands: Inventory, beginning Add: Purchases Less: Inventory, ending Cost of sales, unadjusted Add: to adjust to net realizable value the items in ending inventory Less: Item sold under repurchase agreement Cost of sales, adjusted

P19,450 127,850 8,500 138,800 200 6,000 P133,000

7. Inventory per count on December 31, 2012 as reported by WS Company was at P560,000. The amount does not include the following items: A. Merchandise costing P27,500 received on January 3, 2013, and purchase invoice recorded on January 5, 2013. Shipment was made on December 25, 2012,terms FOB destination. B. Merchandise costing P20,000 was physically segregated in the warehouse and was set aside for shipmentto a customer (shipment scheduled on January 2, 2013). This will be invoiced to the customer by then at P28,000. C. Merchandise received on January 7, 2013, costing P17,000 was entered in the purchases journal on January 8, 2013. Invoice shipment was made FOB shipping point on December 30, 2012. What is the adjusted balance of inventories to be presented in the 2012 statement of financial position? A P597,000 . B. P580,000 C. P578,000 D P587,500 . Solution: A is correct Page 4 of 45

Unadjusted inventory balance Item (a) Goods received January 2013, under FOB Destination, thus item is correctly excluded Item (b) Goods still in warehouse but were not included in the count Item (c) Goods received January 2013, under FOB shipping point. Items shipped December 30, 2012 Adjusted inventory balance

P560,000 20,000 17,000 P597,000

8. Bohol Company leased office premises to Jill Company for a 5-year term starting January 2, 2013. Under the terms of the lease, rent for the first year is P200,000 and rent for years 2 through 5 is P300,000 annually. As an inducement to enter the lease, Bohol Company waives the first six month payments. Jill Company likewise paid a P70,000 security deposit of which 80% is refundable at the end of the lease term. Furthermore, contingent rent equal to 2% of sales in excess of P12,000,000 shall be paid by Jill Company. Bohol Company incurred initial direct cost of P40,000 while JL Company paid P30,000 in costs in relation to the lease. In 2013, Jill Company reported sales of P13,000,000. Rental expense to be included in Jill Company’s 2013 income statement is A 288,800 . B. 268,800 C. 302,200 D 315,000 . Solution: A is correct Total rental payments for five years(P200,000/2) + (P300,000 x 4) Divided by total years Annual rental expense (fixed) Contingent rent [(P13M – P12M) x 2%] Direct cost incurred(30,000 / 5) Non-refundable portion of security deposit(P70,000 x 20%) / 5 Total rental expense

P1,300,000 5 years P260,000 20,000 6,000 2,800 P288,800

The following information applies to the next two questions: Listed below are four interbank cash transfers, indicated by the numbers 1, 2, 3, and 4, of a client for late December 2010 and early January 2011: Bank Account One Bank Account Two Disbursing Date

Receiving Date

(Month/Day)

(Month/Day)

Per Bank

Per Books

Per Bank Per Books

1.

12/31

12/30

12/31

12/30

2.

1/2

12/30

12/31

12/31

3.

1/3

12/31

1/21/2

4.

1/3

12/31

1/212/31

Page 5 of 45

9.

Based on the schedule of interbank transfers above, which of the cash transfers indicates an error in cash cutoff at December 31, 2010? a.

1

b.

2

c.

3

d.

4

Answer: C

10.

Based on the schedule of interbank transfers above, which of the cash transfers would appear as a deposit in transit on the December 31, 2010 bank reconciliation? a.

1

b.

2

c.

3

d.

4

Answer: D Auditing Theory 1. Which of the following are the three principal methods in sample selection are specifically identified in PSA 530? A. Statistical sampling, attribute sampling and haphazard sampling B. Random number selection, systematic selection and haphazard selection C. Sequential sampling, discovery sampling and statistical sampling D. Sequential sampling, discovery sampling and random number selection Answer: B (PSA 530, Audit Sampling) 2. Which of the following is an appropriate consideration in auditor’s selection of sample size? A. The auditor may select a voided or cancelled document in a sample. If the document has been properly voided, treat the item as deviation. B. If the auditor encounters missing documents and he is unable to determine whether control has been properly performed, replace the document with another sample item. C. The auditor may select a voided or cancelled document in a sample. If the document has been properly voided, replace the document with another sample item D. If the auditor encounters missing documents and he is unable to determine whether control has been properly performed, treat the item as deviation and replace the document with another sample item. Answer: C (PSA 530, Audit Sampling) Page 6 of 45

The auditor may select a voided or cancelled document in a sample. If the document has been properly voided, the document must be replaced with another sample item. If the auditor encounters missing documents and he is unable to determine whether control has been properly performed, the auditor must treat the item as deviation. 3. Fraudulent financial reporting involves intentional misstatements or omissions of amount or disclosures in the financial statements to deceive financial statement users. Which of the following acts appropriately involves fraudulent financial statements? I. II. III. IV. V. A. B. C. D.

Manipulations, falsification or alteration of records or documents Lapping of accounts receivable Recording of transactions without substance Stealing entity’s assets such as cash, marketable securities and inventory Intentional application of accounting policies I and II I, II , III and IV I, III and V All of the above

Answer: C (PSA 240, Auditor’s Responsibility to Consider Fraud in an Audit of Financial Statements) Fraudulent financial reporting involves intentional misstatements or omissions of amount or disclosures in the financial statements to deceive financial statement users. This may involve: I. Manipulations, falsification or alteration of records or documents. II. Misrepresentation in or intentional omission of the effects of transactions from records or documents. III. Recording of transactions without substance and IV. Intentional application of accounting policies 4. Which of the following is not true with reference to the Code of Professional Ethics for CPAs? A. Acting as advocate of an audit client in resolution of a dispute or litigation that involves amounts material to the financial statements of the audit client impairs CPA’s independence. B. A firm or network firm should not provide accounting and bookkeeping services for an audit client that is not a listed entity. C. Fees are not to be regarded as contingent if these are fixed by a court D. A professional accountant in public practice may enter into an arrangement for the purchase of the whole or part of an accounting practice requiring payments to individuals formerly engaged in the practice. Answer: B (Code of Professional Ethics for CPAs) According to Code of Professional Ethics for CPAs, a firm or network firm should not provide accounting and bookkeeping services for an audit client that is a listed entity. The provision of such services may impair CPAs independence, or at least give the appearance of impairing independence. 5. Which of the following acts is prohibited by the Code of Professional Ethics for CPAs? A. The use of a firm name which includes the name of a retired partner B. An announcement in a newspaper of the opening of a public accounting office C. Engaging in civic activities during business hours D. Accepting an engagement or employment which one cannot reasonable expect to compete or discharge with professional competence

Page 7 of 45

Answer: D (Code of Professional Ethics for CPAs) 6. When an independent auditor’s examination of financial statements discloses special circumstances that make the auditor suspects that fraud may exist, the auditor’s initial course of action should be to A. Recommend that the client pursue the suspected fraud to a conclusion that is agreeable to the auditor B. Extend normal audit procedures in an attempt to detect the full extent of the suspected fraud C. Reach an understanding with the proper client representative as to whether the auditor or the client is to make the investigation necessary to determine if a fraud has in fact occurred D. Determine whether the fraud, if in fact it does exist, might be of such a magnitude as to affect the auditor’s report on the financial statements Answer: D (PSA 250, Consideration of Laws and Regulations in an Audit of Financial Statements, Section 58) 7. When an independent auditor decides that the work performed by internal auditors may have a bearing on the nature, timing and extent of planned audit procedures, the independent auditor should evaluate the objectivity of the internal auditors. Relative to objectivity, the independent auditor should A. B. C. D.

Consider the organizational level to which internal auditors report Review the quality control program in effect for the internal audit staff Examine the quality of the internal audit reports Consider the qualifications of the internal audit staff

Answer: A (PSA 610, Using the Work of Internal Auditors) 8. During an assessment of the risk associated with sales contracts and related commissions, which of the following factors would most likely result in an expansion of the engagement scope? A. An increase in product sales, along with an increase in commissions B. An increase in sales returns, along with an increase in commissions C. A decrease in sales commissions, along with a decrease in product sales D. A decrease in sales returns, along with an increase in product sales Answer: B (Gleim’s Certified Internal Auditor Reviewer) Trends described in A, C and D are compatible while trend in B are incompatible and may indicate inflated sales figures. 9. Which of the following explanations suggests the least amount of relative risk stemming from a failure to compare a purchase order to an approved price list? A. A temporary employee processed the purchase order. B. The comparison is not required by company policy. C. The vendor is one used often by the company. D. The director of the purchasing department approved the purchase order. Answer: A (Sawyer’s Certified Internal Auditor Reviewer) “A” is correct. The relative risk of loss to the company would be lower if a temporary employee were responsible for the error. This suggests an isolated incident, as the employee will not be able to repeat the error once he or she has left the purchasing department. However, the error should still be reported to management. Page 8 of 45

“B” is incorrect. If employees are not required to consult an approved price list, the company risks buying at too great a price. “C” is incorrect. Even if the company regularly buys from this vendor, there is no assurance that the price is reasonable unless the purchase order is compared to the approved price list. “D” is incorrect. The director should have required the purchase order to be compared to the approved price list before approving it. Otherwise, the purchase price may exceed the approved price.

A. B. C. D.

10. Which of the following fraudulent entries is most likely to be made to conceal the theft of an asset? Debit expenses, and credit the asset Debit the asset, and credit another asset account Debit revenue, and credit the asset Debit another asset account, and credit the asset Answer: A (Sawyer’s Certified Internal Auditor Reviewer) “A” is correct. Most fraud perpetrators would attempt to conceal their theft by charging it against an expense account. “B” is incorrect. Debiting the stolen asset account would be going in the wrong direction to conceal an asset theft. “C” is incorrect. An entry decreasing revenue would be unusual and would stand out. “D” is incorrect. This entry would not permanently conceal the fraud. It would simply shift the unreconcilable balance to another asset account. 11. What computer-assisted audit technique would an auditor use to identify a fictitious or terminated employee? A. Parallel simulation of payroll calculations. B. Exception testing for payroll deductions. C. Recalculations of net pay. D. Tagging and tracing of payroll tax-rate changes. Solution: B a) Incorrect. In a parallel simulation, data that were processed by the engagement client’s system are reprocessed through the auditor’s program to determine if the output obtained matches the output generated by the client’s system. This technique might identify problems with the client’s processing but would not identify a fictitious or terminated employee. b) Correct. This type of computer-assisted audit technique (CAAT) program can identify employees who have no deductions. This is important because fictitious or terminated employees will generally not have any deductions. c) Incorrect. A CAAT program can recalculate amounts such as gross pay, net pay, taxes and other deductions, and accumulated or used leave times. These recalculations can help determine if the payroll program is operating correctly or if employee files have been altered, but would not identify a fictitious or terminated employee. d) Incorrect. In this type of CAAT program, certain actual transactions are “tagged,” and as they proceed through the system, a data file is created that traces the processing through the system and permits an auditor to subsequently review that processing. This would not, however, identify a fictitious or terminated employee. 12. A chief audit executive (CAE) suspects that several employees have used desktop computers for personal gain. In conducting an investigation, the primary reason that the CAE chose to engage a forensic information systems auditor rather than using the organization’s information systems auditor is that a forensic information systems auditor would possess: A. Knowledge of the computing system that would enable a more comprehensive assessment of the computer use and abuse. Page 9 of 45

B. Knowledge of what constitutes evidence acceptable in a court of law. C. Superior analytical skills that would facilitate the identification of computer abuse. D. Superior documentation and organization skills that would facilitate in the presentation of findings to senior management and the board. Solution: B a) Incorrect. The organization’s information systems auditor would probably have more knowledge of the organization’s computing systems. b) Correct. The distinguishing characteristic of forensic auditing is the knowledge needed to testify as an expert witness in a court of law. Although a forensic auditor may possess the other attributes listed, the organization’s information systems auditor may also possess these skills or knowledge elements. c) Incorrect. A forensic auditor would not necessarily have analytical or organizational skills that are superior to those of the organization’s auditor. d) Incorrect. See answer “c”. 13. An audit committee is concerned that management is not addressing all internal audit observations and recommendations. What should the audit committee do to address this situation? A. Require managers to provide detailed action plans with specific dates for addressing audit observations and recommendations. B. Require all managers to confirm when they have taken action. C. Require the chief executive officer to report why action has not been taken. D. Require the chief audit executive to establish procedures to monitor progress. Solution: D a) Incorrect. Management are responsible for ensuring action on all internal audit observations and recommendations, but some actions may take time to complete and it is not practical to expect that all will be resolved when an audit committee meets. b) Incorrect. See answer “a”. c) Incorrect. See answer “a”. d) Correct. The chief audit executive is responsible for establishing appropriate procedures for monitoring the progress by management on all internal audit observations and recommendations. This responsibility should be written into its charter by the audit committee, and progress should be reported at each audit committee meeting. 14. As part of a manufacturing company’s environmental, health, and safety (EHS) self-inspection program, inspections are conducted by a member of the EHS staff and the operational manager for a given work area or building. If a deficiency cannot be immediately corrected, the EHS staff member enters it into a tracking database that is accessible to all departments via a local area network. The EHS manager uses the database to provide senior management with quarterly activity reports regarding corrective action. During review of the self-inspection program, an auditor notes that the operational manager enters the closure information and affirms that corrective action is complete. What change in the control system would compensate for this potential conflict of interest? A. No additional control is needed because the quarterly report is reviewed by senior management, providing adequate oversight in this situation. B. No additional control is needed because those implementing a corrective action are in the best position to evaluate the adequacy and completion of that action. C. After closure is entered into the system, review by the EHS staff member of the original inspection team should be required in order to verify closure. D. The EHS department secretary should be responsible for entering all information in the tracking system based on memos from the operational manager. Page 10 of 45

Solution: C a) Incorrect. Although senior management can use the report to question why certain corrective actions may be behind schedule, they have no way of knowing whether the corrective actions shown as complete were actually completed. b) Incorrect. While the operational managers may in fact be the most knowledgeable about the corrective action, independent verification is preferable. c) Correct. If there is a step in the process at which someone independent of the area being inspected can evaluate the adequacy and completeness of corrective action, the potential for closure fraud is minimized. d) Incorrect. There is nothing inappropriate about the environmental, health, and safety staff entering the initial inspection results. Having the secretary enter closure data does not improve controls since there is still no independent review. It is also less efficient and timely than having the data entered directly in the field. 15. An audit of an organization’s claims department determined that a large number of duplicate payments had been issued due to problems in the claims processing system. During the exit conference, the vice president of the claims department informed the auditors that attempts to recover the duplicate payments would be initiated immediately and that the claims processing system would be enhanced within six months to correct the problems. Based on this response, the chief audit executive should: A. Adjust the scope of the next regularly scheduled audit of the claims department to assess controls within the claims processing system. B. Monitor the status of corrective action and schedule a follow-up engagement when appropriate. C. Schedule a follow-up engagement within six months to assess the status of corrective action. D. Discuss the findings with the audit committee and ask the committee to determine the appropriate followup action. Solution: B a) Incorrect. Because the finding is significant, the internal audit activity should not wait until the next regularly scheduled audit to assess the status of corrective action. b) Correct. The internal audit activity should monitor the status of the corrective action. A follow-up engagement should be scheduled when changes to the claims processing system have been sufficiently completed to allow for testing of adequacy and effectiveness. c) Incorrect. Although management indicated that the corrections should be completed within six months, this may not be the case. As a result, the internal audit activity should monitor the status of corrective action and schedule a follow-up engagement when it is appropriate. d) Incorrect. Although the findings should be discussed with the audit committee because of their significance, the scope and timing of a follow-up engagement should be determined by the chief audit executive based on available information. 16. An organization uses electronic data interchange and on-line systems rather than paper-based documents for purchase orders, receiving reports, and invoices. Which of the following audit procedures would an auditor use to determine if invoices are paid only for goods received and at approved prices? A. Select a statistical sample of major vendors and trace the amounts paid to specific invoices. B. Use generalized audit software to select a sample of payments and match purchase orders, invoices, and receiving reports stored on the computer using a common reference. C. Select a monetary-unit sample of accounts payable and confirm the amounts directly with the vendors. D. Use generalized audit software to identify all receipts for a particular day and trace the receiving reports to checks issued.

Page 11 of 45

Solution: B A. Incorrect. This procedure only provides data on whether payments agree with invoices. It does not provide data on whether the invoiced amounts are correct. B. Correct. This would help the auditor determine that all three pieces of data were most likely matched before payment. C. Incorrect. As with answer “a”, this only provides data on whether payments agree with invoices. It does not provide data on whether the goods were actually received. D. Incorrect. This provides data only on one day. While it matches items received with those paid, it does not provide data on whether the billings were correct. 17. A sales department has been giving away expensive items in conjunction with new product sales to stimulate demand. The promotion seems successful, but management believes the cost may be too high and has asked for a review by the internal audit activity. Which of the following procedures would be the least useful to determine the effectiveness of the promotion? A. Comparing product sales during the promotion period with sales during a similar non-promotion period. B. Comparing the unit cost of the products sold before and during the promotion period. C. Performing an analysis of marginal revenue and marginal cost for the promotion period, compared to the period before the promotion. D. Performing a review of the sales department’s benchmarks used to determine the success of a promotion. Solution: B a) Incorrect. This comparison would help highlight the effectiveness of the promotion in increasing sales. b) Correct. There is no indication that cost of the products sold has changed. The challenge is to address the effectiveness of the promotion. c) Incorrect. This is the key analysis as it would show the extent of additional revenue versus cost. d) Incorrect. This would be helpful because the sales department may have useful information on new customers and repeat purchases. 18. Which of the following observations by an auditor is most likely to indicate the existence of control weaknesses over safeguarding of assets? I. II. III. IV.

a) b) c) d)

A service department's location is not well suited to allow adequate service to other units. Employees hired for sensitive positions are not subjected to background checks. Managers do not have access to reports that profile overall performance in relation to other benchmarked organizations. Management has not taken corrective action to resolve past engagement observations related to inventory controls. I and II only. I and IV only. II and III only. II and IV only.

Solution: D (II and IV only) I. II. III.

Incorrect. This is a symptom of weak controls for achieving organizational goals and objectives, but not for safeguarding of assets. Correct. This is a symptom of weak controls for safeguarding of assets. Incorrect. This is a symptom of weak controls for achieving organizational goals and objectives, but not for safeguarding of assets. Page 12 of 45

IV.

Correct. Management's failure to take corrective action on past engagement observations, which related to safeguarding of assets, is a weakness related to safeguarding of assets.

19. The use of an analytical review to verify the correctness of various operating expenses would not be a preferred approach if: A. An auditor notes strong indicators of a specific fraud involving these accounts. B. Operations are relatively stable and have not changed much over the past year. C. An auditor would like to identify large, unusual, or non-recurring transactions during the year. D. Operating expenses vary in relation to other operating expenses, but not in relation to revenue. Solution: A a) Correct. If the auditor already suspects fraud, a more directed audit approach would be appropriate. b) Incorrect. Relatively stable operating data is a good scenario for using analytical review. c) Incorrect. Analytical review would be useful in identifying whether large, non-recurring, or unusual transactions occurred. d) Incorrect. Analytical review only needs to have accounts related to other accounts or other independent data. It does not require that they be related to revenue. 20. As part of a preliminary survey of the purchasing function, an auditor read the department's policies and procedures manual. The auditor concluded that the manual described the processing steps well and contained an appropriate internal control design. The next engagement objective was to determine the operating effectiveness of internal controls. Which procedure would be most appropriate in meeting this objective? a) b) c) d)

Prepare a flowchart. Prepare a system narrative. Perform a test of controls. Perform a substantive test.

Solution: C a) Incorrect. Flowcharts are most appropriate for studying internal control design. The audit objective is whether the controls are in place and effective, which indicates the need for a test of controls. b) Incorrect. System narratives are most appropriate for studying internal control design. The audit objective is whether the controls are in place and effective, which indicates the need for a test of controls. c) Correct. Tests of controls, also known as compliance tests, help an auditor determine whether controls are being followed and are effective. For instance, a policy may require that all large transactions be approved by a manager. As a test of controls, the auditor may sample large transactions and review whether manager approval was obtained and whether the proposed transaction meets all the criteria that the manager was supposed to verify. d) Incorrect. Substantive tests are tests to determine whether an objective has been achieved and do not necessarily test internal controls. Management Accounting Services 1. Product cost under absorption costing is characteristically: A . B. C.

Higher than under variable costing. Lower than under variable costing. Equal to variable costing. Page 13 of 45

D .

Higher sometimes and lower sometimes than variable costing.

Solution: A is correct. 2. NFJPIA Co. is considering the following investment proposals. Rank the following proposals using the profitability index (in order of the most profitable proposal down to the least profitable proposal). Investment required Present value of future net cash flows A . B. C. D .

A 160,000 192,000

B 200,000 300,000

C 120,000 168,000

D 150,000 240,000

D, B, C, A B, D, C, A B, D, A, C A, C, B, D

Solution: A is correct. Profitability index is an investment appraisal technique calculated by dividing the present value of future cash flows of a project by the initial investment required for the project. Profitability index of the above proposals are as follows: Decision Rule. Accept a project if the profitability index is greater than 1, stay indifferent if the profitability index is zero and don't accept a project if the profitability index is below 1. The ranking of the proposals in terms of preference using the profitability index is determined as follows.

Present value of future net cash flows ÷ Investment required Profitability index

A 192,000 160,000 1.2

B 300,000 200,000 1.5

C 168,000 120,000 1.4

D 240,000 150,000 1.6

Page 14 of 45

3. The stock of GVS pays dividends that are expected to grow at a steady rate of 3.2% per year. Investors expect a rate of return of 11.5% from GVS stock. If the spread between this required rate of return and the dividend growth rate were to increase by 50 basis points, the percentage change in the stock price would be: A . B. C. D .

+3.94% -4.73% -5.68% +4.22%

Solution: C is correct. The dividend discount model (DDM) is a method of valuing a company based on the theory that a stock is worth the discounted sum of all of its future dividend payments. In other words, it is used to value stocks based on the net present value of the future dividends. Using Dividend Discount Model given Po = D1/(k – g). When k – g = 11.5% - 3.2% = 8.3%, the price is given by Po = D1/0.083. When the spread increases by 50 basis points and all else stays constant, the price becomes P1= D1/(0.083 + 0.005) = D1/0.088. The percentage change in the price equals (P1 – Po)/Po = (1/0.088 – 1/0.083)*0.083 = 0.083/0.088 – 1 = -5.68%. Thus, the stock price falls by 5.68% when the spread between k and g increases by 50 basis points, all else equal. 4. Which of the following statements is true? A . B. C. D .

A firm with financial leverage has a larger equity multiplier than an otherwise identical firm with no debt in its capital structure. The use of debt in a company’s capital structure results in tax benefits to the investors who purchase the company’s bonds. All else equal, a firm with a higher debt ratio will have a lower basic earning power ratio. All of the statements above are correct.

Solution: A is correct. A firm with financial leverage means the firm uses debt for financing. Equity multiplier is a measure of financial leverage, calculated as: Total Assets / Total Stockholders' Equity. Like all debt management ratios, the equity multiplier is a way of examining how a company uses debt to finance its assets. It is also known as the financial leverage ratio or leverage ratio. A higher equity multiplier indicates higher financial leverage, which means the company is relying more on debt to finance its assets. The use of debt provides tax benefits to the corporations that issue debt, not to the investors who purchase debt (in the form of bonds). The basic earning power ratio is calculated as the ratio of Earnings Before Interest and Taxes to Total Assets. The higher the BEP ratio, the more effective a company is at generating income from its assets. BEP is useful for comparing firms with different tax situations and different degrees of financial leverage. The basic earning power ratio would be the same if the only thing that differed between the firms were their debt ratios. Choice A is correct. Other choices are false.

Page 15 of 45

5. The issuance of serial bonds in exchange for an office building, with the first installment of the bonds due late this year: A . B. C. D .

Decreases net working capital Decreases the current ratio Decreases the quick ratio Affects all of the answers as indicated

Solution: D is correct D is correct because the first installment is a current liability which affects the quick ratio, the current ratio, and working capital. 6. The internal rate of return is the A . B. C. D .

Rate of interest that equates the present value of cash outflows and the present value of cash inflows Minimum acceptable rate of return for a proposed investment Risk-adjusted rate of return Required rate of return

Solution: A is correct. 7. An analyst has obtained the following information regarding two companies, Company X and Company Y:      

Company X and Company Y have the same total assets. Company X has a higher interest expense than Company Y. Company X has a lower operating income (EBIT) than Company Y. Company X and Company Y have the same return on equity (ROE). Company X and Company Y have the same total assets turnover (TATO). Company X and Company Y have the same tax rate.

Based on the above information, which of the following statements is true? A . B. C. D .

Company X has a higher times interest earned ratio. Company X and Company Y has the same debt ratio. Company X has a higher return on assets. Company X has a lower profit margin.

Solution: D is correct. Times interest earned (also called interest coverage ratio) is the ratio of earnings before interest and tax (EBIT) of a business to its interest expense during a given period. It is a solvency ratio measuring the ability of a business to pay off its debts. X has a lower net income considering it has a lower EBIT and higher interest than Y, but the same tax rate as Y. Sales for each company are the same because they have the same total assets and the same total assets turnover ratio (TATO = Sales/TA). Therefore, since X has a lower net income and same sales as Y, it follows that it has a lower profit margin (Net Income /Sales). Page 16 of 45

8. Which of the following statements relating to breakeven point analysis is correct? Statement 1 Statement 2 Statement 3 Statement 4 A . B. C. D . E.

Breakeven point is not a good planning tool since the goal of business is to make a profit. Focusing on target net income rather than operating income will increase the breakeven point. Companies with a greater proportion of fixed costs have a greater risk of loss than companies with a greater proportion of variable costs. Breakeven point is the level of sales at which profit is zero.

False, False, True, True False, True, True, True True, True, False, False True, False, False, False All statements are true.

Solution: A is correct. Statement 1 is false - breakeven point is an important planning tool that helps managers determine volume of sales/production needed to be profitable. Statement 2 is false - the same breakeven point will be calculated whether target operating income or target net income is used in the CVP calculation. Statements 3 and 4 are correct. 9. A call option on a share of common stock is more valuable when there is lower A . B. C. D .

Market value of the underlying share Exercise price on the option Time to maturity on the option Variability of market price on the underlying share

Answer: B. Exercise price on the option. Source: Wiley CPA Exam Review – Business Environment and Concepts by Whittington and Delaney (2005) 10. Which of the following will cause a shift in the supply curve of a product? A . B. C. D .

Changes in consumer tastes Changes in the number of buyers in the market Changes in production taxes Changes in the price of the product

Answer: C. Changes in production taxes A shift in the supply curve may result from: Page 17 of 45

(1) (2) (3) (4) (5) (6) A . B. C . D .

changes in production technology changes or expected changes in resource prices changes in the prices of other goods changes in taxes or subsidies changes in the number of sellers in the market expectations about the future price of the product

Incorrect. Changes in consumer tastes will result in a shift in demand curve. Incorrect. Changes in the number of buyers in the market will result in a shift in demand curve. Correct. Changes in production taxes will alter the supply curve. Incorrect. Change in the price of the product involves movement along the existing supply curve, not a shift in the supply curve.

Source: Wiley CPA Exam Review – Business Environment and Concepts by Whittington and Delaney (2005) 11. Operational budgets are used by a retail company for planning and controlling its business activities. Data regarding the company’s monthly sales for the last 6 months of the year and its projected collection patterns are shown below. The cost of merchandise averages 40% of its selling price. The company’s policy is to maintain an inventory equal to 25% of the next month’s forecasted sales. The inventory balance at cost is Php80,000 as of June 30. Forecasted sales July August September October November December

Php775,000 750,000 825,000 800,000 850,000 900,000

What is the budgeted cost of the company’s purchase for the month of August? A. B. C. D.

Php307,500 Php300,000 Php450,500 Php142,500

Answer: A. Php307,500 Sales (Php750,000 × 40%) Ending inventory (Php825,000 × 40% × 25%) Beginning inventory (Php750,000 × 40% × 25%) Purchases

Php300,000 82,500 (75,000) Php307,500

Source: Wiley CPA Exam Review – Business Environment and Concepts by Whittington and Delaney (2005) 12. The management of ABC Corporation has decided to implement a transfer pricing system. ABC’s MIS department is currently negotiating a transfer price for its services with the four producing divisions of the company as well as the marketing department. Charges will be assessed based on number of reports (assume that all reports require the same amount of time and resources to produce). The cost to operate the Page 18 of 45

MIS department at its full capacity of 1,000 reports per year is budgeted at Php450,000. The user subunits expect to request 250 reports each this year. The cost of temporary labor and additional facilities used to produce reports beyond capacity is budgeted at Php480.00 per report. ABC could purchase the same services from an external Information Services firm for Php700,000. What amounts should be used as the floor and the ceiling in determining the negotiated transfer price? Floor Ceiling A. Php450 Php500 B. Php456 Php560 C. Php540 Php500 D. Php500 Php540 Answer: B. Floor Php456.00; Ceiling Php560.00 Negotiated transfer prices should fall within a range limited by a ceiling and a floor. The ceiling is the lowest market price that could be obtained from an external supplier, and the floor equals the outlay costs plus opportunity cost of the transferring division. Since ABC’s MIS department does not have the option to sell services to external customers, its opportunity cost is Php0. Since all costs of service departments must be covered by the revenue-producing departments, the MIS department’s outlay cost equals its total costs. The department’s full capacity level is 1,000 reports per year. However, the user departments will be requesting 1,250 reports (5 user subunits × 250 reports each). Thus, the MIS department will incur costs of Php120,000 [Php480 × (1,250 – 1,000)] for the 250 reports above capacity, in addition to the Php450,000 budgeted costs for full capacity. The total cost of Php570,000 (Php450,000 + Php120,000) is used to calculate the floor. The ceiling is based on the Php700,000 that would be incurred to purchase MIS services externally. Since the MIS department will be producing 1,250 reports, the floor is Php456.00 (Php570,000 ÷ 1,250), and the ceiling is Php560.00 (Php700,000 ÷ 1,250). At full capacity, any differential costs of additional production are added to the floor. Php480.00 represents only the differential cost of producing each report above full capacity, not cost per report for total production. Budgeted costs are based on production of 1,250 reports, not 1,000. Source: Wiley CPA Exam Review – Business Environment and Concepts by Whittington and Delaney (2005) 13. ABC Company has a single product called JGH. The company normally produces and sells 70,000 JGHs each year at a selling price of Php45.00 per unit. The company’s unit costs at this level of activity are given below: Direct materials Direct labor Variable manufacturing overhead Fixed manufacturing overhead Variable selling expenses Fixed selling expenses Total cost per unit

Php10.00 3.50 6.80 8.00 2.00 3.00 Php33.30

Assume that ABC Company has sufficient capacity to produce 100,000 units of JGH each year without any increase in fixed manufacturing overhead costs. The company could increase its sales by 30% above the present 70,000 units each year if it were willing to increase the fixed selling expenses by Php100,000. How much would be the increase in income if the production and sales are increased by 30%? A. B. C. D.

Php376,700 Php300,000 Php465,600 Php356,750

Answer: A. Php376,700 Selling price

Php45.00 Page 19 of 45

Variable expenses Direct materials Direct labor Variable manufacturing overhead Variable selling expenses Unit contribution margin

Php10.00 3.50 6.80 2.00

22.30 Php22.70

Additional contribution (70,000 units × 30% × Php22.70) Less: Additional fixed selling costs Additional income

Php476,700 100,000 Php376,700

Source: Comprehensive Review in MAS (2007) by Bobadilla 14. Combo Music Company produces and sells sensitive precision instruments for industrial use. Four types of instruments are produced at Canlubang Plant. Data with respect to these four types of products are given below: Product Line

Time required to make such unit (in minutes) Selling price per unit Variable cost per hour

1

2

3

4

20 24 108

40 34 48

10 16 72

60 40 20

The Company has 24,000 production hours available after meeting the requirements for the regular market. The market can absorb all units of any product line that is produced with one exception - only 48,000 units of Product 3 can be sold. In addition, there is a shortage of materials for Product 4. Materials are available for the production of a total of 12,000 units of Product 4. What is the number of hours available for Product 2 to maximize profit while observing all the restrictions? A 6,000 hours . B. 4,000 hours C. 3,000 hours D 0 hours . Answer: B. 4,000 hours.

Total minutes in an hour Divided by: Minutes to make each unit Units produced each hour Multiplied by: Unit selling price Sales per hour Variable cost per hour Contribution margin per hour

1 60 20 3 24 72 108 (36)

Product Line 2 3 60 60 40 10 1.5 6 34 16 51 96 48 72 3 24

4 60 60 1 40 40 20 20

Page 20 of 45

Product ranking of production and sales to achieve the best product combination and thus maximize profit: Product 1 Product 2 Product 3 Product 4

will not be manufactured 3rd 1st 2nd

Allocation of 24,000 production hours: 1 Product 3 – (48,000 × 10/60) Product 4 Product 2 – Remainder Product 1 – None Total allocated hours

Product Line 2 3 8,000

4 12,000

4,000 0

4,000

8,000

12,000

Source: Reviewer in MAS by Morante 15. Which of the following statements regarding transfer pricing is false? A . B. C. D .

When idle capacity exists, there is no opportunity cost to producing intermediate products for another division. Market-based transfer prices should be reduced by any costs avoided by selling internally rather than externally. No contribution margin is generated by the transferring division when variable cost-based transfer prices are used. The goal of transfer pricing is to provide segment managers with incentive to maximize the profits of their divisions.

Solution: D is correct The goal of transfer pricing is to encourage managers to make transfer decisions which maximize profits of the company as a whole. Some transfers may not be profitable to a particular division, but would effect a cost savings to the company by avoiding costs of purchasing externally. For example, when a division is already operating at full capacity and uses variable cost transfer prices, additional production for internal transfer would result in a loss for the transferring division because no contribution margin is earned to cover the differential fixed costs incurred. Conversely, internal production may be cheaper to the corporate entity than purchasing the product, in which case the division should accept the order. However, the division manager is likely to engage in suboptimization by rejecting the order to enhance the division’s performance, while adversely affecting overall company performance. Business Law and Taxation 1. Marsh offered to construct the house of Mellow for Php900,000, giving the latter ten (10) days within which to accept or reject the offer. On the fifth day, before Mellow could make up his mind, Marsh withdrew his offer. Which of the following is true? A. The withdrawal of Marsh’s offer will cause the offer to cease in law, hence, there could be no concurrence of the offer and the acceptance; B. In the absence of concurrence of offer and acceptance, there can be no consent; C. Without consent, there is no perfected contract for the construction of the house of Mellow. D. All of the above. Answer: d) All of the above. Article 1318 of the Civil Code – “There is no contract unless the following requisites concur: Page 21 of 45

(1) Consent of the contracting parties; (2) Object certain which is the subject matter of the contract; (3) Cause of the obligation which is established. Article 1319 of the Civil Code – “Consent is manifested by the meeting of the offer and the acceptance upon the thing and the cause which are to constitute the contract. The offer must be certain and the acceptance absolute. A qualified acceptance constitutes a counter offer. Xxx.” 2. W, X, and Y are partners. X conveyed his share to Z. Thereafter, the partnership incurred liabilities for the contract entered into between the partnership and M prior to the sale of X’s share to Z. Which of the following statements is correct? A. Z cannot be held liable to M because a partner admitted into a partnership shall become liable for an obligation only from the time he took part in the control of the business. B. Z cannot be held liable to M because as a partner, Z shall be subject to all the restrictions and liabilities of a partnership. C. Z can be held liable pro-rata with the other partners regardless of the time the liability was incurred. D. Z can be held liable to M, but his liability shall be satisfied only out of partnership property, unless there is a stipulation to the contrary. Answer: D Z can be held liable to M, but his liability shall be satisfied only out of partnership property, unless there is a stipulation to the contrary. Article 1826 of the Civil Code provides: “A person admitted as a partner into an existing partnership is liable for all the obligations of the partnership arising before his admission as though he had been a partner when such obligations were incurred, except that his liability shall be satisfied only out of partnership property, unless there is a stipulation to the contrary.” 3. ABC owns 15% of the subscribed capital stock of XYZ corporation which owns an office building. ABC owns a security and janitorial agency. ABC, a director of XYZ Corporation, engaged the services of his own company to perform services to the office building. Which of the following conditions is NOT required in order for the service contract between ABC and XYZ to be valid? A. The service contract must be approved by the board of directors of XYZ corporation; B. ABC’s vote was not necessary to constitute a quorum at the meeting of the board of directors; C. ABC’s vote was not necessary for the approval of the service contracts. D. The service contract must be ratified by all the stockholders of the corporation. Answer: D The service contract must be ratified by all the stockholders of the corporation. Section 32 of the Corporation Code provides: “A contract of the corporation with one or more of its directors or trustees or officers is voidable, at the option of such corporation, unless all the following conditions are present: 1. That the presence of such director of trustee in the board meeting in which the contract was approved was not necessary to constitute a quorum for such meeting; 2. That the vote of such director or trustee was not necessary for the approval of the contract; 3. That the contract is fair and reasonable under the circumstances; and 4. That in the case of an officer, the contract has been previously authorized by the board of directors. Where the first two conditions set forth in preceding paragraph is absent, in case of a contract with a director or trustee, such contract may be ratified by the vote of the stockholders representing at least two-thirds of the outstanding capital stock or of at least two thirds of the members in a meeting called for the purpose: Provided, That full disclosure of the adverse interest of the directors or trustees involved is made at such meeting: Provided, however, That the contract is fair and reasonable under the circumstances.” 4. In line with the Trust Fund Doctrine that generally renders it unlawful for the corporation to return assets to the stockholders representing capital, under what conditions may a stock corporation acquire its own shares: A. There exists in the corporate books unrestricted retained earnings to cover the repurchase of shares; B. The repurchase of shares must be for a legitimate business purpose; Page 22 of 45

C. Both A and B. D. Neither A nor B. Answer: C Both A and B. Section 41 of the Corporation Code provides: “ A stock corporation shall have the power to purchase or acquire its own shares for a legitimate corporate purposes or purposes, including but not limited to the following cases: Provided, That the corporation has unrestricted retained earnings in its books to cover the shares to be purchased or acquired: a. To eliminate fractional shares arising out of stock dividends; b. To collect or compromise an indebtedness to the corporation, arising out of unpaid subscription, in a delinquency sale, and to purchase delinquent shares sold during said sale; and c. To pay dissenting or withdrawing stockholders entitled to payment for their shares under the provisions of this Code. 5. In a stockholders’ meeting, Mr. Bean dissented from the corporate act converting preferred voting shares to non-voting shares. Mr. Bean submitted his certificates of stock for notation that his shares are dissenting. The next day, Mr. Bean transferred his shares to Ms. Dora to whom new certificates were issued. Which of these statements is correct? A. Mr. Bean can exercise the right of appraisal because he dissented from the corporate act. B. Mr. Bean can no longer exercise the right of appraisal because he already transferred his shares to Ms. Dora. C. Ms. Dora should exercise the right of appraisal on behalf of Mr. Bean. D. None of the statements is correct. Answer: B. Mr. Bean can no longer exercise the right of appraisal because he already transferred his shares to Ms. Dora. Section 86 of the Corporation Code provides: “Within ten (10) days after demanding payment for his shares, a dissenting stockholder shall submit the certificates of stock representing his shares to the corporation for notation thereon that such shares are dissenting shares. His failure to do so shall, at the option of the corporation, terminate his rights under this Title. If shares represented by the certificates bearing such notation are transferred, and the certificates consequently cancelled, the rights of the transferor as a dissenting stockholder under this Title shall cease and the transferee shall have all the rights of a regular stockholder; and all dividend contributions which would have accrued on such shares shall be paid to the transferee.” 6. ABC Corporation has 10 members in the Board of Directors. During the election of officers, one of its members got sick, while the other two went abroad, thus, the three of them failed to attend the meeting and cast their vote. Nonetheless, A acquired four votes for the position of Secretary. Is A validly elected as an officer? A. No. The Code requires the vote of majority of all the members of the Board of Directors. B. Yes. The Code requires only the decision of at least a majority of directors present at the election. C. Yes. The articles of incorporation or by-laws may provide for lesser majority in case of election of officers. D. No. The articles of incorporation require the quorum. Answer: A. No. The Code requires the vote of majority of all the members of the Board of Directors. Section 25 of the Corporation Code: “Immediately after their election, the directors of a corporation must formally organize by the election of a president, who shall be a director, a treasurer who may or may not be a director, a secretary who shall be a resident and citizen of the Philippines, and such other officers as may be provided for in the by-laws. Any two or more positions may be held concurrently by the same person, except that no one shall act as president and secretary or as president and treasurer at the same time. The directors or trustees and officers to be elected shall perform the duties enjoined on them by law and by the bylaws of the corporation. Unless the articles of incorporation or the by-laws provide for a greater majority, a Page 23 of 45

majority of the number of directors or trustees as fixed in the articles of incorporation shall constitute a quorum for the transaction of corporate business, and every decision of at least a majority of the directors or trustees present at a meeting at which there is a quorum shall be valid as a corporate act, except for the election of officers which shall require the vote of a majority of all the members of the board. Directors or trustees cannot attend or vote by proxy at board meetings.” 7. A, B, C, D, and E entered into a contract of partnership to organize events such as weddings and birthday celebrations. A and B are the only limited partners. During one of the parties that the partnership organized, D totally forgot her task to turn on the lights during the entrance of the debutant. As a result, the debutant slipped off injuring her ankle. The victim sues the partnership for damages. Choose the best answer. A. A and B are not liable because they are limited partners. B. The partnership is liable for damages to the same extent as D. C. Only the general partners are liable in equal proportion. D. Only D is liable. Answer: B. The partnership is liable for damages to the same extent as D. Article 1822 of the Civil Code provides: “Where, by any wrongful act or omission of any partner acting in the ordinary course of the business of the partnership or with the authority of his co-partners, loss or injury is caused to any person, not being a partner in the partnership, or any penalty is incurred, the partnership is liable therefor to the same extent as the partner so acting or omitting to act.” 8. X owes Y Php100,000 payable in one year, at 10% interest per annum. On maturity, X pays Y Php50,000 which Y acknowledged having received without mentioning the payment of interest. Which of the following is correct? A. Principal has already been paid. B. Interest is waived. C. Payment will be applied first to the interest. D. Obligation is extinguished. Answer: C. Payment will be applied first to the interest. Article 1253 of the Civil Code provides: “If the debt produces interest, payment of the principal shall not be deemed to have been made until the interests have been covered.” 9. ABC Corporation is a stock corporation whose principal place of business is in Quezon City. If a regular meeting of its stockholders shall be held, one of the requirements is: A. Notice of meetings must be published in a newspaper of general circulation two weeks before the scheduled meeting. B. Quorum must consist of all the stockholders. C. Written notice of meetings shall be sent at least two weeks prior to the meeting, unless a different period is required by the by-laws. D. All of the above. Answer: C. Written notice of meetings shall be sent at least two weeks prior to the meeting, unless a different period is required by the by-laws. Section 50 of the Corporation Code provides: “Regular meetings of stockholders or members shall be held annually on a date fixed in the by-laws, or if not so fixed, on any date in April of every year as determined by the board of directors or trustees: Provided, That written notice of regular meetings shall be sent to all stockholders or members of record at least two (2) weeks prior to the meeting unless a different period is required by the by-laws.

Page 24 of 45

Special meetings of stockholders or members shall be held at any time deemed necessary or as provided in the bylaws; Provided, however, That at least one (1) week written notice shall be sent to all stockholders or members, unless otherwise provided in the by-laws. Xxx.” 10. XYZ Corporation’s by-laws provide that A, one of its directors, shall receive 10% of the net income before tax of the corporation as his yearly compensation. However, B, the Corporation’s president, contends that such provision directly violates the Code prohibiting the compensation of directors. Are directors absolutely not entitled to compensation? A. Yes. The Code expressly provides that directors shall not receive any compensation, except for reasonable per diems. B. No. Directors are entitled to compensation when authorized by the by-laws or by vote of stockholder representing at least majority of the outstanding capital stock, subject to limitations that the total yearly compensation must exceed 10% of the net income before income tax of the compensation during the preceding year. C. No. Directors are entitled when authorized by the by-laws or by vote of stockholders representing at least majority of the outstanding capital stock, subject to limitations that the total yearly compensation shall not exceed 10% of the net income before tax of the corporation during the preceding year. D. Yes. Directors are not entitled to compensation. Answer: C. No. Directors are entitled when authorized by the by-laws or by vote of stockholders representing at least majority of the outstanding capital stock, subject to limitations that the total yearly compensation shall not exceed 10% of the net income before tax of the corporation during the preceding year. Section 30 of the Corporation Code provides: “Compensation of directors. -- In the absence of any provision in the by-laws fixing their compensation, the directors shall not receive any compensation, as such directors, except for reasonable per diems: Provided, however, That any such compensation other than per diems may be granted to directors by the vote of the stockholders representing at least a majority of the outstanding capital stock at a regular or special stockholders’ meeting. In no case shall the total yearly compensation of directors, as such directors, exceed ten percent (10%) of the net income before income tax of the corporation during the preceding year.” 11. Employee J, a resident Filipino citizen, is a teacher in Monsters University. He earns a monthly salary of P25,000. For the year 2012, he also received a discretionary bonus amounting to P40,000. His wife, K, gave birth to their fourth child. The names of their children and the birthdays are as follows: Ethel Luz Edmon d Grace

3/11/199 1 6/14/199 8 6/19/200 6 9/15/201 2

For the year ended 2012, how much basic personal exemption can Employee J claim? A. B. C. D. E.

P150,000 P125,000 P100,000 P50,000 Zero

Answer: D. P50,000 Page 25 of 45

12. Co. C had the following data in the month of September: Sales Purchases of goods Purchase of machines Machine Life

P1,900,000 1,260,000 1,440,000 8 years

Assuming that the said transactions are properly substantiated for VAT purposes, compute the VAT payable. Note: The amounts are exclusive of VAT. A. Php70,920 B. Php76,800 C. Php73,920 D. Php225,120 Answer: C. P73,920 228,000

Output VAT (P1,900,000 x 12%) Input VAT Purchase of goods (P1,260,000 x 12%) Purchase of machines* (P1,440,000 x 12%/60 months) Net VAT Payable

151,200 2,880

154,080 73,920

13. On July 31, 2013, Esperanza received a preliminary assessment notice from the BIR demanding that she pays P180,000.00 deficiency income taxes on her 2011 income. How many days from July 31, 2013 should Esperanza respond to the notice? A. 180 days B. 30 days C. 60 days D. 15 days Answer: D. 15 days 14. The gross estate of Ms. Layda amounted to P5 million. The actual funeral expenses are as follows: Mourning apparel of the surviving spouse Mourning apparel of married children Publication charges for death notices Interment and/or cremation fees and charges Total

P100,000 90,000 70,000 50,000 310,000

How much is the total deductible funeral expense? A. P250,000 B. P200,000 C. P270,000 D. P180,000 Answer: B. P200,000 Actual funeral expenses Mourning apparel of the surviving spouse Publication charges for death notices Interment and/or cremation fees and charges

100,000 70,000 50,000 220,000 Page 26 of 45

5% of gross estate

250,000

Limit

200,000

15. Gianna Corporation is engaged in the business of publishing magazines as well as the merchandising of office supplies. For CY 2012, it has the following revenue/sales: Revenue from magazines Sales from office supplies

587,400 197,500

Further, Gianna incurred entertainment, amusement and recreation expenses (EAR) amounting to P8,350 and 2/3 of which was incurred for the publishing of magazines. How much EAR is deductible for income tax purposes? A. B. C. D.

P8,657.33 P8,350.00 P6,861.50 P6,554.17

Answer: D. P6,554.17 Revenue

Actual EAR

Revenue from magazines

587,400

2/ 3

5,56 6.67

Sales from office supplies

197,500

1/ 3

2,78 3.33 8,35 0.00

EAR LIMIT (1% of 5,874.0 revenue from 0 magazines) (0.5% of sales 987.5 from office 0 supplies)

Deductible EAR 5,566.6 7 987.5 0 6,554.1 7

Theory of Accounts 1. This refers to IASB’s main vehicle for consulting the public and sets out a specific proposal in the form of a proposed IFRS (or amendment to an IFRS). A. Final Standards B. Due Process Handbook C. Amendments D. Exposure Draft Answer: D. Exposure Draft Due Process Handbook for the IASB and IFRS Interpretations Committee (February 2013) provides: An Exposure Draft is the IASB’s main vehicle for consulting the public and therefore includes an invitation to comment, setting out the issues that the IASB has identified as being of particular interest. Although it is normally included with the ballot draft, it is not necessary for the IASB to ballot the invitation to comment. (Par. 6.3) An Exposure Draft sets out a specific proposal in the form of a proposed Standard (or amendment to a Standard) and is therefore generally set out in the same way as, and has all of the components of, a Standard. (Par. 6.2) Page 27 of 45

2. Which of the following is not an indicator of situations that would individually or in combination lead to a lease being classified as a finance lease? A . B. C. D .

The leased assets are of such a specialized nature that only the lessee can use them without major modifications. Gains or losses from the fluctuation in the fair value of the residual accrue to the lessor. The lessee has the ability to continue the lease for a secondary period at a rent that is substantially lower than market rent. If the lessee can cancel the lease, the lessor's losses associated with the cancellation are borne by the lessee.

Answer: B. Gainsor losses from the fluctuation in the fair value of the residual accrue to the lessor. Paragraphs 10-11 of PAS 17, Leases state: Whether a lease is a finance lease or an operating lease depends on the substance of the transaction rather than the form of the contract.1 Examples of situations that individually or in combination would normally lead to a lease being classified as a finance lease are: (a) the lease transfers ownership of the asset to the lessee by the end of the lease term; (b) the lessee has the option to purchase the asset at a price that is expected to be sufficiently lower than the fair value at the date the option becomes exercisable for it to be reasonably certain, at the inception of the lease, that the option will be exercised; (c) the lease term is for the major part of the economic life of the asset even if title is not transferred; (d) at the inception of the lease the present value of the minimum lease payments amounts to at least substantially all of the fair value of the leased asset; and (e) the leased assets are of such a specialized nature that only the lessee can use them without major modifications. Indicators of situations that individually or in combination could also lead to a lease being classified as a finance lease are: (a) if the lessee can cancel the lease, the lessor's losses associated with the cancellation are borne by the lessee; (b) gains or losses from the fluctuation in the fair value of the residual accrue to the lessee (for example, in the form of a rent rebate equaling most of the sales proceeds at the end of the lease); and (c) the lessee has the ability to continue the lease for a secondary period at a rent that is substantially lower than market rent. 3. Which of the following statement(s) is/are incorrect? Statement I: PAS 17, Leases, permits initial direct costs incurred by lessors in negotiating an operating lease to be charged as expenses when incurred. Statement II: It is mandatory to apply the finance lease accounting methodology set out in PAS 17 to investment properties held under operating leases. A . B. C. D .

None of the above Statement I only Statement II only Statements I and II

Answer: B. Statement I only Paragraph 52 of PAS 17 provides: Page 28 of 45

“Initial direct costs incurred by lessors in negotiating and arranging an operating lease shall be added to the carrying amount of the leased asset and recognized as an expense over the lease term on the same basis as the lease income.” Paragraph IN5 of PAS 17 provides: “Although PAS 40, Investment Property prescribes the measurement models that can be applied to investment properties held, it requires the finance lease accounting methodology set out in this Standard to be used for investment properties held under leases.” Paragraph 25 of PAS 40 provides: “The initial cost of a property interest held under a lease and classified as an investment property shall be as prescribed for a finance lease by paragraph 20 of PAS 17, i.e., the asset shall be recognized at the lower of the fair value of the property and the present value of the minimum lease payments. An equivalent amount shall be recognized as a liability in accordance with that same paragraph.” 4. Which of the following statement(s) regarding impairment testing under PAS 36, Impairment of Assets, is/are correct? Statement I: Value in use is the undiscounted future cash flows expected to be derived from an asset or cash-generating unit. Statement II: When the amount estimated for an impairment loss is greater than the carrying amount of the asset to which it relates, an entity shall automatically recognize a liability. Statement III: If it is not possible to estimate the recoverable amount of an individual asset, the entity should determine the recoverable amount of the cash generating unit to which the asset belongs. Statement IV: PAS 36 requires an intangible asset with an indefinite useful life to be tested for impairment annually by comparing its carrying amount with its recoverable amount, irrespective of whether there is any indication that it may be impaired. However, the most recent detailed calculation of such an asset's recoverable amount made in a preceding period may be used in the impairment test for that asset in the current period as long as certain criteria provided by PAS 36 are met. A . B. C. D .

Statements III and IV Statements II, III and IV Statements I and III Statements I, III and IV

Answer: A. Statements III and IV Par. 6 of PAS 36, Impairment of Assets, provides: “Value in use is the present value of the future cash flows expected to be derived from an asset or cashgenerating unit.” Par. 62 of PAS 36, Impairment of Assets, provides: “When the amount estimated for an impairment loss is greater than the carrying amount of the asset to which it relates, an entity shall recognize a liability if, and only if, that is required by another Standard.” Par. 22 of PAS 36, Impairment of Assets, provides: “Recoverable amount is determined for an individual asset, unless the asset does not generate cash inflows that are largely independent of those from other assets or groups of assets. If this is the case, recoverable amount is determined for the cash-generating unit to which the asset belongs…” Par. 24 of PAS 36, Impairment of Assets, provides: Page 29 of 45

“Paragraph 10 requires an intangible asset with an indefinite useful life to be tested for impairment annually by comparing its carrying amount with its recoverable amount, irrespective of whether there is any indication that it may be impaired. However, the most recent detailed calculation of such an asset's recoverable amount made in a preceding period may be used in the impairment test for that asset in the current period, provided all of the following criteria are met: (a) if the intangible asset does not generate cash inflows from continuing use that are largely independent of those from other assets or groups of assets and is therefore tested for impairment as part of the cashgenerating unit to which it belongs, the assets and liabilities making up that unit have not changed significantly since the most recent recoverable amount calculation; (b) the most recent recoverable amount calculation resulted in an amount that exceeded the asset's carrying amount by a substantial margin; and (c) based on an analysis of events that have occurred and circumstances that have changed since the most recent recoverable amount calculation, the likelihood that a current recoverable amount determination would be less than the asset's carrying amount is remote.” 5. Which of the following are considered adjusting events after the reporting period in accordance with PAS 10, Events after the Reporting Period? I. II. III. IV. V. A . B. C. D .

Bankruptcy of a customer that occurs after the reporting period Decline in market value of investments Declaration of dividends to holders of equity instruments after the reporting period Determination after the reporting period of the amount of profit-sharing or bonus payments, if the entity had a present legal or constructive obligation at the end of the reporting period to make such payments as a result of events before that date Abnormally large change in asset prices or foreign exchange rates

Statements I, IV and V Statements I, II and IV Statements I and IV Statements I, II, IV and V

Answer: C. Statements I and IV Paragraph 9 of PAS 10, Events after the Reporting Period provides the following examples of adjusting events after the reporting period as follows: (a) the settlement after the reporting period of a court case that confirms that the entity had a present obligation at the end of the reporting period. The entity adjusts any previously recognized provision related to this court case in accordance with PAS 37, Provisions, Contingent Liabilities and Contingent Assets or recognizes a new provision. The entity does not merely disclose a contingent liability because the settlement provides additional evidence that would be considered in accordance with paragraph 16 of PAS 37. (b) the receipt of information after the reporting period indicating that an asset was impaired at the end of the reporting period, or that the amount of a previously recognized impairment loss for that asset needs to be adjusted. For example: (i)

the bankruptcy of a customer that occurs after the reporting period usually confirms that a loss existed at the end of the reporting period on a trade receivable and that the entity needs to adjust the carrying amount of the trade receivable; and

(ii)

the sale of inventories after the reporting period may give evidence about their net realizable value at the end of the reporting period.

(c) the determination after the reporting period of the cost of assets purchased, or the proceeds from assets sold, before the end of the reporting period. Page 30 of 45

(d) the determination after the reporting period of the amount of profit-sharing or bonus payments, if the entity had a present legal or constructive obligation at the end of the reporting period to make such payments as a result of events before that date (see PAS 19, Employee Benefits). (e)

the discovery of fraud or errors that show that the financial statements are incorrect.

6. When disclosing financial instruments, an entity shall group them into classes as follows: I. II. III . A . B. C. D

Group the financial instruments into classes that are appropriate to the nature of the information disclosed and that take into account the characteristics of those financial instruments. Group the financial instruments according to the categories of financial instruments in PAS 39, Financial Instruments: Recognition and Measurement. Provide sufficient information to allow reconciliation to the line items presented in the statement of financial position. I only. I and II. I and III I, II and III.

Answer C. I and III are true: A .

B.

C .

D

Incorrect. Although Statement I is correct such that PFRS 7 paragraph 6 provides that an entity shall group financial instruments into classes that are appropriate to the nature of the information disclosed and that take into account the characteristics of those financial instruments, the standard also requires that sufficient information shall be provided to permit reconciliation to the line items presented in the statement of financial position (Statement III). Incorrect. Although Statement I is correct such that PFRS 7 paragraph 6 provides that an entity shall group financial instruments into classes that are appropriate to the nature of the information disclosed and that take into account the characteristics of those financial instruments, grouping the financial instruments into categories (Statement II) is not consistent with the standard’s requirement to group financial instruments into classes. Correct. I and III are true. PFRS 7.6 provides that an entity shall group financial instruments into classes that are appropriate to the nature of the information disclosed and that take into account the characteristics of those financial instruments. Also, the standard requires that sufficient information shall be provided to permit reconciliation to the line items presented in the statement of financial position. Incorrect. Statements I and III are consistent with the requirements in PFRS 7.6 about grouping financial instruments into classes. However, grouping the financial instruments into categories (Statement II) is not consistent with the standard’s requirement to group financial instruments into classes. The correct answer is C. PFRS 7.6 provides when this PFRS requires disclosures by class of financial instrument, an entity shall group financial instruments into classes that are appropriate to the nature of the information disclosed and that take into account the characteristics of those financial instruments. An entity shall provide sufficient information to permit reconciliation to the line items presented in the statement of financial position.

7. Which of the following statement(s) is/are true?

Page 31 of 45

Statement I:

If an entity's interim financial report is in compliance with PAS 34, Interim Financial Reporting, that fact shall be disclosed. An interim financial report shall not be described as complying with PFRSs unless it complies with some or substantially all of the requirements of PFRSs. If an entity publishes a condensed set of financial statements in its interim financial report, the form and content of those statements shall always conform to the requirements of PAS 1, Presentation of Financial Statements. Costs that are incurred unevenly during an entity's financial year shall be anticipated or deferred for interim reporting purposes if, and only if, it is also appropriate to anticipate or defer that type of cost at the end of the financial year.

Statement II: Statement III:

A . B. C. D .

Statement III only. Statements I and II. Statements II and III Statements I, II and III.

Answer: A. Statement III only. PAS 34 provides: “If an entity's interim financial report is in compliance with this Standard, that fact shall be disclosed. An interim financial report shall not be described as complying with PFRSs unless it complies with all of the requirements of PFRSs” (Par.19) “If an entity publishes a complete set of financial statements in its interim financial report, the form and content of those statements shall conform to the requirements of PAS 1 for a complete set of financial statements.” (Par. 9) “Costs that are incurred unevenly during an entity's financial year shall be anticipated or deferred for interim reporting purposes if, and only if, it is also appropriate to anticipate or defer that type of cost at the end of the financial year.” (Par. 39) 8. Which of the following statement(s) is/are incorrect in accordance with PAS 28, Investment in Associates and Joint Ventures? Statement I: Statement II:

A . B. C. D .

If an investment in an associate becomes an investment in a joint venture, the entity continues to apply the equity method and does not remeasure the retained interest. When downstream transactions provide evidence of a reduction in the net realizable value of the assets to be sold or contributed, or of an impairment loss of those assets, those losses shall be recognized to the extent of the investor’s interest in the associate.

All of the above. Statement I only Statement II only None of the above.

Answer: C. Statement II only PAS 28 states that:

Page 32 of 45

Paragraph 24 - “If an investment in an associate becomes an investment in a joint venture or an investment in a joint venture becomes an investment in an associate, the entity continues to apply the equity method and does not remeasure the retained interest.” Paragraph 29 - “When downstream transactions provide evidence of a reduction in the net realizable value of the assets to be sold or contributed, or of an impairment loss of those assets, those losses shall be recognized in full by the investor…” 9. Under the PAS 19, Employee Benefits (Revised), the following are components of defined benefit costs under defined benefit plans, except: A . B. C. D .

Service cost in profit or loss Net interest on the net defined benefit liability in profit or loss Remeasurements of the net defined benefit liability (asset) in other comprehensive income Remeasurements of the net defined benefit liability (asset) in profit or loss

Answer: D. Remeasurements of the net defined benefit liability (asset) in profit or loss Par. 120 of PAS19R states: “An entity shall recognize the components of defined benefit cost, except to the extent that another PFRS requires or permits their inclusion in the cost of an asset, as follows: (a) service cost (see paragraphs 66-112) in profit or loss; (b) net interest on the net defined benefit liability (asset) (see paragraphs 123-126) in profit or loss; and (c) remeasurements of the net defined benefit liability (asset) (see paragraphs 127-130) in other comprehensive income.” 10. Which of the following is/are an example of a situation in which offsetting is inappropriate? Statement I: Financial assets and financial liabilities that arise from financial instruments having the same primary risk exposure (e.g., assets and liabilities within a portfolio of forward contracts or other derivative instruments) but involve different counterparties. Statement II: Financial or other assets pledged as collateral for non-recourse financial liabilities Statement III: Obligations incurred as a result of events giving rise to losses are expected to be recovered from a third party by virtue of a claim made under an insurance contract. Statement IV: The entity currently has a legally enforceable right to set-off the recognized amounts and intends either to settle on a net basis, or to realize the asset and settle the liability simultaneously. A . B. C. D .

Statement I only Statements I and II only Statements I, II and II I only Statements I, II, III and IV

Answer: C. Statements I, II and III only Paragraph 49 of PAS 32, Financial Instruments: Presentation, states that the conditions set out in paragraph 42 of PAS 32 are generally not satisfied and offsetting is usually inappropriate when: (a) several different financial instruments are used to emulate the features of a single financial instrument (a 'synthetic instrument'); Page 33 of 45

(b) financial assets and financial liabilities arise from financial instruments having the same primary risk exposure (for example, assets and liabilities within a portfolio of forward contracts or other derivative instruments) but involve different counterparties; (c) financial or other assets are pledged as collateral for non-recourse financial liabilities; (d) financial assets are set aside in trust by a debtor for the purpose of discharging an obligation without those assets having been accepted by the creditor in settlement of the obligation (for example, a sinking fund arrangement); or (e) obligations incurred as a result of events giving rise to losses are expected to be recovered from a third party by virtue of a claim made under an insurance contract. Practical Accounting 1 1. WXY Company has an overdue 8% note payable to First Bank at Php8,000,000 and accrued interest of Php640,000. As a result of a restructuring agreement entered on December 31, 2014, First Bank agreed to the following: a. b. c. d.

The principal obligation is reduced to Php7,000,000. The accrued interest of Php640,000 will be waived. The date of maturity is extended to December 31, 2018. Annual interest of 10% is to be paid in 4 years every December 31 starting in 2015.

The present value of 1 at 8% for 4 periods is 0.735 and the present value of an ordinary annuity of 1 at 8% for 4 periods is 3.31. How much is the gain on extinguishment to be recognized for 2014 assuming that the prevailing market rate for similar note is at 8%? A. B. C. D.

P1,178,000 P538,000 P1,641400 P0

Solution: A is correct Original principal Accrued interest Carrying amount of overdue debt and interest Restructured principal @ present value (Php7,000,000 × 0.735) Restructured interest @ present value (Php7,000,000 × 10% × 3.31) Gain on extinguishment (13.6% of the carrying amount of overdue debt and interest)

Php8,000,000 640,000 Php8,640,000 5,145,000 2,317,000

7,462,000 Php 1,178,000

Par. 40 of PAS 39, Financial Instruments: Recognition and Measurement, provides: “… a substantial modification of the terms of an existing financial liability or a part of it (whether or not attributable to the financial difficulty of the debtor) shall be accounted for as an extinguishment of the original financial liability and the recognition of a new financial liability.” Par. 41 of PAS 39 provides: “The difference between the carrying amount of a financial liability (or part of a financial liability) extinguished or transferred to another party and the consideration paid, including any non-cash assets transferred or liabilities assumed, shall be recognized in profit or loss.” Page 34 of 45

Par. 62 of Application Guidance of PAS 39 provides: “For the purpose of paragraph 40, the terms are substantially different if the discounted present value of the cash flows under the new terms, including any fees paid net of any fees received and discounted using the original effective interest rate, is at least 10 per cent different from the discounted present value of the remaining cash flows of the original financial liability. If an exchange of debt instruments or modification of terms is accounted for as an extinguishment, any costs or fees incurred are recognized as part of the gain or loss on the extinguishment. If the exchange or modification is not accounted for as an extinguishment, any costs or fees incurred adjust the carrying amount of the liability and are amortized over the remaining term of the modified liability.” 2. ABC Company has five million ordinary shares in issue at the beginning of Year 1. In the middle of Year 2 it announces a rights issue whereby all existing shareholders will be entitled to buy one share for every five they hold, at a price of 50. Immediately prior to the issue, the share price was 80. The profits for Years 1, 2, and 3 were 200 million, 300 million, and 360 million, respectively. The rights were exercised immediately upon issuance. What are the (restated) basic earnings per share for each of these three years? A . B. C. D .

Year 1: 40.00, Year 2: 52.93, Year 3: 60.00 Year 1: 40.00, Year 2: 54.55, Year 3: 60.00 Year 1: 37.49, Year 2: 52.93, Year 3: 60.00 Year 1: 37.49, Year 2: 54.55, Year 3: 60.00

Solution: C is correct EPS as originally calculated in Year 1 will be 40 (200 million / 5 million). In the following year, it is necessary to calculate the bonus element of the rights issue. This is done as follows: Computing the theoretical ex-rights value per share Fair value of all outstanding shares before exercise of rights + Total amount received from exercise of rights –––––––––––––––––––––––––––––––––––––––––––––––––––––––––––––––––––––– Number of shares outstanding before exercise of rights + Number of shares issued in exercise of rights 400,000,000 + 50,000,000 = –––––––––––––––––––––––

= 75

5,000,000 + 1,000,000

Computing the adjustment factor Fair value per share prior to exercise of rights –––––––––––––––––––––––––––––––––––––

= 80 / 75 = 1.067

Theoretical ex-rights value per share

Computing basic earnings per share Page 35 of 45

Year 1 as originally reported 40.00 Year 1 as restated (200,000,000 / [5,000,000 x 1.067]) = 37.49 Year 2 300,000,000 –––––––––––––––––––––––––––––––––

=

52.93

(5,000,000 x 1.067 x 0.5) + (6,000,000 x 0.5) Year 3 360,000,000 ––––––––––––

60.00

6,000,000  3. LKF Company has the following outstanding liabilities and capital as of December 31, 2012: a. Three million ordinary shares issued at P1.00 (which is also the par value) three years ago. LKF Company has bought back 2,500 of its shares this year. b. P6,500,000 million loan with GHC Bank maturing August 31, 2013. The company has communicated with GHC Bank that it will roll over the said loan on November 7, 2012. Upon rollover, the maturity of the said loan is deferred to August 31, 2014. This was noted by the bank. c. Accounts payable amounting to P3,489,000. This account does not include the following items: (1) Purchases from suppliers amounting to P678,000 shipped FOB shipping point. The goods were in transit as of December 31, 2012. (2) Purchases from suppliers amounting to P58,000 shipped FOB destination. The goods were in transit as of December 31, 2012. d. Dividends on cumulative preference shares amounting to P3,500,000. The declaration of the said dividends was made only on February 6, 2013. e. P2,890,000 purchase commitment entered during 2012 to buy 57,800 kilos of corn to be delivered next year. f. P7,590,000 million loan with JHZ Bank maturing September 30, 2013. The company has a credit line with KLM Bank and plans to drawdown Php7,590,000 million from KLM Bank on September 30, 2013 to pay off the loan with JHZ Bank. The new loan will mature on September 30, 2016. g. Deferred tax liabilities as of December 31, 2012 amounted to Php5,673,000. h. P9,000,000 convertible debt with VCD Company maturing June 30, 2013. The Company may either settle the debt through payment or issuance of ordinary shares. i. P1,000,000,000 redeemable preference shares. The preference shares were issued five years ago. The company is required to redeem the shares on or before May 31, 2013. How much is the current liability to be presented in the balance sheet as of December 31, 2012? A . B. C. D .

P1,023,647,000 P1,011,757,000 P1,014,647,000 P1,020,757,000

Solution: D is correct c.

Php3,489,000 Page 36 of 45

c.1 f. h. i. Total current liabilities

678,000 7,590,000 9,000,000 1,000,000,000 Php1,020,757,000

4. On January 1, 2014, KGY Company contracted with a contractor to construct an equipment for P43,000,000. The equipment was completed during the year. KGY Company is required to make the following payments in 2012: January 1 March 1 July 1 September 1 December 1 Total

P4,300,000 21,500,000 5,000,000 7,600,000 4,600,000 P43,000,000

KGY Company had the following outstanding debt as of December 31, 2013 and December 31, 2014: 11% 4-year note with simple interest payable annually on March 31 (specifically used to fund the construction) 12% 5-year loan with simple interest payable annually on June 30 10% 3-year loan with simple interest payable annually on July 31

P15,000,000 20,000,000 11,000,000

How much interest should be capitalized by KGY Company for the year ended December 31, 2014? A. B. C. D.

P4,854,700 P3,076,303 P5,150,000 P3,092,170

Solution: B is correct Date January 1 March 1 July 1 September 1 December 1

Expenditure Fraction 4,300,000 12/12 21,500,000 10/12 5,000,000 6/12 7,600,000 4/12 4,600,000 1/12 Average expenditures

Average expenditures Applicable to specific borrowing Applicable to general borrowings

Average P4,300,000 17,916,667 2,500,000 2,533,333 383,333 P27,633,333 P27,633,333 (15,000,000) P12,633,333

12% 5-year loan with simple interest payable annually on June 30 10% 3-year loan with simple interest payable annually on July 31 General borrowings Average capitalization rate (3,500,000 ÷ 31,000,000) Interest on specific borrowing (15,000,000 × 11%)

Principal P20,000,000 11,000,000 P31,000,000

Interest P2,400,000 1,100,000 P3,500,000

11.29% P1,650,000 Page 37 of 45

Interest on general borrowing (12,633,333 × 11.29%) Total capitalizable interest

1,426,303 P3,076,303

5. JKL Company, on adoption of PAS 41, Agriculture, has reclassified certain assets as biological assets in its 2011 Statement of Financial Position. The total value of the entity’s biological assets as of December 31, 2013 was P2,000,000 comprising: Fruit-bearing trees Land under trees Roads near the plantation

P1,700,000 200,000 100,000

On February 28, 2014, the owner of the entity purchased P300,000 worth of farm animals from his friend to be used in the business. However, such animals do not have a quoted market price, and is not traded in an active market. Other valuation methods are also clearly inappropriate or unworkable. The estimated lives of the trees and the animals are 20 years and 5 years, respectively. During the last quarter of the year 2014, the following events occurred: Increase in fair value in 2012 due to growth of trees Decrease in fair value due to harvest Harvest of fruits Estimated fair value of the animals based on the judgment of the entity’s CPA

P100,000 90,000 500,000 500,000

As of the December 31, 2014 Statement of Financial Position, how much should be presented as Biological Assets by JKL Company? A . B. C. D .

P1,960,000 P2,010,000 P2,210,000 P1,950,000

Solution: A is correct Fair value of fruit bearing trees, December 31, 2013 Decrease in fair value due to harvest Increase in fair value to due to growth of trees Fair value of fruit bearing trees, December 31, 2014 Farm animals, February 28, 2014 Depreciation of farm animals (P300,000/5) × 10/12 Farm animals, December 31, 2014 Fruit bearing trees Farm animals Total

P1,700,000 (90,000) 100,000 P1,710,000 P300,000 (50,000) P250,000 P1,710,000 250,000 P1,960,000

6. Lex Company shipped inventory on consignment to Lionel Company that cost P20,000. Lionel Company paid P500 for advertising that was reimbursable from Lex Company. At the end of the year, 70% of the inventory was sold for P30,000. The agreement states that a commission of 20% will be provided to Lionel Company for all sales. What amount of inventory on consignment remains in the balance sheet of Lex Company? Page 38 of 45

A. 0

B. 4,800

C. 6,000

D. 6,150

Solution: C is correct Cost of inventory sent out on consignment 20,000 Multiply by: % of goods in the hand of consignee 30% Inventory on consignment 6,000 C 7. On January 1, 2014, AJ Corporation purchased several pieces of inventory for P20,000. However, SC Company, the seller agreed to wait for exactly two years before receiving payment. Then, on December 31, 2014, AJ Company sells the entire said inventory to BY Corporation for P30,000. AJ Company agrees to wait for exactly three years to receive the P30,000 payment. A reasonable interest rate for these transactions is 8% although no separate cash interest is to be paid on either the purchase or the sale. The gross profit that AJ Company should recognize for the year ended December 31, 2014 is (Round off present value factors to 5 decimal places): A. 10,000

B.

7,938

C.

6,668

D.

3,815

Solution: C is correct Sales Less: Cost of sales Gross Profit

30,000 x PVF P1 @ 8% for 3 periods 20,000 x PVF P1 @ 8% for 2 periods

0.79383 0.85734

23,815 17,147 6,668 C

8. In December 2014, Judith Company began including one coupon in each package of candy that it sells and offers a toy in exchange for P5 and 5 coupons. The toys cost P12 each and an additional P4 to deliver it to customers. Eventually, 80% of the coupons will be redeemed. During December, Judith Company sold 220,000 packages of candy, 70,000 coupons were already sent for redemption of which 20,000 is still under processing by year end. The estimated liability in the December 31, 2014 balance sheet of Judith Company is A. 233,200

B.

277,200

C.

330,000

D. 387,200

Solution: B is correct Total coupons issued Multiply by: Coupons expected to be redeemed Less: coupons received and processed (70,000 - 20,000) Remaining coupons expected to be redeemed (and processed) in the future Divide by: conversion rate toys/coupons Expected number of toys to be given away Multiply by: cost per unit (12 + 4 -5) Total estimated liability 12/31/10

220,000 80% 176,000 50,000 126,000 5 25,200 11 277,200 B

9. Barry Company purchased equipment by making a down payment of P4,000 and issuing a note payable for P18,000. A payment of P6,000 is to be made at the end of each year for three years. The applicable rate of interest is 8%. Shipping charges for the equipment were P2,000, and installation charges were P3,500. How much is the capitalized cost of the equipment (Round off present value factors to 2 decimal places)? A. 19,480

B. 21,480

C. 24,980

D. 27,500 Page 39 of 45

Solution: C is correct Down payment Add: PV of future installments @ 8% Shipping cost Installation cost Capitalized cost of equipment

(6,000 x 2.58)

4,000 15,480 2,000 3,500 24,980

10. D Company had the following deferred tax balances at reporting date - Deferred tax assets, P1,200,000; Deferred tax liabilities, P3,000,000. Effective from the first day of the next financial period, the company rate of income tax was reduced from 40% to 30%. The adjustment to income tax expense to recognize the impact of the tax rate change is: A. Dr 600,000

B. Cr 600,000

C. Dr 450,000

D. Cr 450,000

Solution: D is correct Tax rate DTA 40% 1,200,000 30% 900,000 Net change (decrease) Practical Accounting 2

DTL 3,000,000 2,250,000

Net DTL amount = 1,800,000 = 1,350,000 (450,000)(D)

1. George Company works on a Php70 million contract in 2014 to construct an office building. During 2014, George Company uses the cost to cost method. As of December 31, 2014, the balances in certain accounts were: Construction in Progress, Php24.5 million; Accounts Receivable, Php2.4 million; and Progress Billings, Php12 million. The estimated future costs to complete the project totaled Php31.85 million as of December 31, 2014. The actual cost incurred in 2014 was A Php17.00 million . B. Php24.50 million C. Php17.15 million D Php70.00 million . Solution: C is correct Construction in Progress Total Contract Price Percentage of completion Estimated future costs to complete Percentage yet to be completed Total estimated cost Estimated future costs to complete Actual cost incurred in 2014

Php24.5 million 70.0 million 35% Php31.85 million 65% 49.00 million 31.85 million Php17.15 million

2. Presented below are items taken from the unadjusted trial balances of Progressive Company and its branch on December 31, 2014. Home Office Books Branch Books Shipments to branch Php300,000 Page 40 of 45

Allowance for overvaluation of branch inventory Shipments from home office Purchases (from outsiders) Merchandise inventory, January 1 Merchandise inventory, December 31 Sales Expenses

99,900 Php390,000 144,600 54,600 48,750 540,000 51,000

It is the company’s policy to bill all branches for merchandise shipments at 30% above cost. How much of the branch inventory on January 1 represents purchases from outsiders? A. Php11,700 B. Php42,000 C. Php42,900 D. Php11,000 Solution: A is correct Allowance for overvaluation of branch inventory, December 31 Over allowance related to shipments during the year Allowance for overvaluation of branch inventory, January 1 Merchandise inventory, January 1 Shipments from home office (9,900/30%*130%) Merchandise inventory purchased from outsiders

Php9,900 90,000 Php9,900 Php54,600 42,900 Php11,700

3. On January 2, 2014, Polo Corporation purchased 80 percent of Seed Company’s common stock for Php216,000. Polo Corporation has assessed that it obtained control over Seed Company in accordance with PFRS 10, Consolidated Financial Statements. Php30,000 of the excess of the consideration paid over the book value of the net assets of Seed Company is attributed to a depreciable asset with an economic life of ten years and the remaining balance to goodwill. On the date of acquisition, Seed reported common stock outstanding of Php80,000 and retained earnings of Php140,000, and Polo reported common stock outstanding of Php350,000 and retained earnings of Php520,000. For the year ended December 31, 2014, Seed reported net income of Php35,000 and paid dividends of Php15,000. Polo reported earnings from its separate operations of Php95,000 and paid dividends of Php46,000. Goodwill had been impaired by Php2,000 on December 31, 2014. How much is the consolidated net income attributable to owners of Polo on December 31, 2014? A. Php118,500 B. Php118,000 C. Php126,150 D. Php126,000 Solution: B is correct Purchase price Proportionate share of NCI in book value of net assets of Seed (20% x 220,000) Less book value of net assets of Seed Company Excess of purchase price and NCI over the book value of net assets of Seed Less fair value adjustment on the acquired depreciable asset Difference attributable to goodwill Net income of Polo Share in net income of Seed (80% x 35,000)

Php216,000 44,000 220,000 40,000 30,000 Php10,000 Php95,000 28,000 Page 41 of 45

Amortization of excess of fair value over book value of the depreciable asset (30,000/10) Impairment of goodwill Consolidated net income attributable to owners of Polo

(3,000) (2,000) Php118,000

4. The Tommy Store began operations by selling wholesale merchandise on an installment basis and uses the installment method of accounting. Terms include down payment of 20% and balance payable in three years; 50% in the year of sale; 30% in the year after; and 20% in the third year. Tommy includes a 25% mark-up on cost for its selling price. Installment sales reported by Tommy are Php550,000 in 2010, Php770,000 in 2013, and Php908,000 in 2014. How much is the installment accounts receivable and the unrealized gross profit at the end of 2014? A Php486,400; Php97,280 . B. Php574,400; Php121,600 C. Php97,280; Php486,400 D Php121,600; Php574,400 . Solution: A is correct 2013 Php770,000*80%*20% 2014 Php908,000*80%*50% Installment receivable, end of 2012 Unrealized gross profit (Php486,400/125%*25%)

Php123,200 363,200 Php486,400 Php97,280

5. Yam Company has used a traditional cost accounting system to apply quality control costs uniformly to all products at a rate of 14.5% of direct labor cost. Monthly direct labor cost for Atswete make-up is Php27,500. In an attempt to distribute quality control costs more equitably, the company is considering activity-based costing. The monthly data shown in the chart below have been gathered for Atswete make-up: Activity

Cost Driver

Incoming material inspection Type of material In-process inspection Number of units Product certification Per order

Cost Rates Php11.50 per type Php0.14 per unit Php77 per order

Quantity for Atswete 12 types 17,500 units 25 orders

The monthly quality control cost assigned to Atswete make-up using activity-based costing is how much higher or lower than the cost using the traditional system? A Php8,500.50 higher than the cost using the traditional system . B. Php525.50 lower than the cost using the traditional system C. Php8,500.50 lower than the cost using the traditional system D Php525.50 higher than the cost using the traditional system . Solution: D is correct Traditional Costing (Php27,500 x 14.5%) Activity Based Costing Incoming material inspection (Php11.50 x 12) In-process inspection (Php0.14 x 17,500) Product certification (Php77 x 25) Difference

Php3,987.50 Php138.00 2,450.00 1,925.00

Php4,513.00 (Php525.50) Page 42 of 45

6. C and D are partners sharing profits and losses in the ratio of 6:4, respectively. On January 1, the partners decided to admit T as a new partner upon his investment of Php96,000. On this date, the interest in the partnership of C and D are as follows: C, Php138,000; D, Php111,600. Assuming that the new partner is given 1/4 interest in the firm. The agreed capital of the partnership is Php360,000. The admission of a new partner will result to which of the following: A. Goodwill is Php20,400. B. Bonus from D is Php2,400. C. Bonus to T is Php6,000. D. Capital balance of C after admission of T to the partnership is Php150,240. Solution: D is correct

Capital before admission of T Investment by T Goodwill Bonus to old partners Capital after admission of T

C Php138,000

D Php111,600

8,640 3,600 Php150,240

5,760 2,400 Php119,760

T (25%) Php– 96,000 (6,000) Php90,000

Total Php249,600 96,000 14,400 – Php360,000

7. The Rockwell Company of Makati opened a branch at Cebu on January 1, 2014 to expand the market of its product. Merchandise shipped during 2014 to the Cebu branch totaled Php59,000, and this included a profit of 25% based on cost. At the end of the year, the inventory was Php6,000 at billed price. Sales on account, Php72,500; Expenses, Php16,500, of which Php1,200 were unpaid on December 31, 2014; Cash received from customers on account, Php40,000, after allowing cash discounts of Php1,470; Cash remitted to the home office during the year, Php33,000. How much is the income of the branch in 2014 insofar as the home office is concerned? A Php13,600 . B. Php12,130 C. Php1,530 D Php3,000 . Solution: B is correct Sales Cash discounts Cost of goods sold [(Php59,000-Php6,000)/125%] Expenses Branch income

Php72,500 (1,470) (42,400) (16,500) Php12,130

8. HIJ Company filed a voluntary bankruptcy petition on September 30, 2014 and the statement of affairs reflects the following amounts: Assets Assets pledged with fully secured creditors Assets pledged with partially secured creditors Free assets Liabilities Liabilities with priority Fully secured creditors

Carrying Amount

Fair Value

Php910,000 511,875 1,137,500

Php1,080,625 341,250 796,250

113,750 739,375 Page 43 of 45

Partially secured creditors Unsecured creditors

568,750 1,478,750

Assume that the assets are converted into cash at the estimated fair values and the business is liquidated. How much should partially secured creditors receive? A Php341,250 . B. Php511,875 C. Php477,750 D Php568,750 . Solution: C is correct Assets pledged with partially secured creditors Share in free assets [60% x (568,750 - 342,250)] Total cash to be received by partially secured creditors

Php341,250 136,500 Php477,750

Excess assets pledged with fully secured creditors (1,080,625 - 739,375) Free assets Total free assets Less liabilities with priority Net free assets Total unsecured liabilities (568,750 - 342,250 + 1,478,750) Expected recovery rate

Php341,250 796,250 1,137,500 (113,750) 1,023,750 1,706,250 60.0%

9. ABC Company, engaged in furniture business around the globe, received an order from a British Corporation under the term of FOB shipping point on December 1, 2014. This was billed for 98,000 UK pounds and payable on January 31, 2015. The furniture was shipped and invoiced on December 12, 2014. This shipment was received by the customer on December 25, 2014. Relevant spot exchange rates for UK pound to Php on various dates follow: December 1, 2014 December 12, 2014 December 25, 2014 December 31, 2014 January 31, 2015

Buying Spot Rate Php68.79 68.82 68.75 69.01 68.95

Selling Spot Rate Php68.85 68.91 68.80 69.11 69.02

How much is the foreign exchange gain or loss to be reported on this transaction in 2014 income statement of ABC Company assuming that the functional currency of ABC Company is Philippine Peso? A Php18,620 gain . B. Php8,820 loss C. Php19,600 loss D Php19,600 gain . Solution: A is correct Buying spot rate as of December 31, 2012 Less: Buying spot rate as of December 12, 2012 Difference

Php69.01 68.82 0.19 Page 44 of 45

Multiply: Amount billed Foreign exchange gain

£98,000 Php18,620

10. Government Agency AAA issued purchase order for the acquisition of office equipment costing Php30,000. The equipment was received with the charge invoice and was paid by check after withholding taxes of 10%. Government Agency AAA remitted the tax withheld to BIR thru a government depository bank. What is the entry of Government Agency AAA to record payment? A . B. C.

D .

Accounts payable Cash – National Treasury – MDS Office equipment Cash – National Treasury – MDS Office equipment Due to BIR Cash – National Treasury – MDS Accounts payable Due to BIR Cash – National Treasury – MDS

Answer: D Accounts payable . Due to BIR Cash – National Treasury – MDS

27,000 27,000 30,000 30,000 30,000 3,000 27,000 30,000 3,000 27,000 30,000 3,000 27,000

Page 45 of 45

View more...

Comments

Copyright ©2017 KUPDF Inc.
SUPPORT KUPDF